« Vrati se
(a) Find the rearrangement \{a_1, \dots , a_n\} of \{1, 2, \dots, n\} that maximizes
a_1a_2 + a_2a_3 + \cdots + a_na_1 = Q.

(b) Find the rearrangement that minimizes Q.

Slični zadaci

#NaslovOznakeRj.KvalitetaTežina
1240IMO Shortlist 1966 problem 570
1259IMO Shortlist 1967 problem 31
1580IMO Shortlist 1982 problem 40
1583IMO Shortlist 1982 problem 70
1586IMO Shortlist 1982 problem 100
1591IMO Shortlist 1982 problem 150